Can someone help me please​

Can Someone Help Me Please

Answers

Answer 1
1. 96%
2. 57.142858%
3. 83.333333333333%

Related Questions

44735758 x 46639756-435333 divided by 48883 = what pls help will give brainleist

Answers

Answer:

2.0864648e+15

Step-by-step explanation:

Answer:

Ok this is a long one 4,792,801,917.519956

Hope this helps! But is this for a class or were you just wondering?

The cost of 5 pounds of boudin is $26.What is the constant of proportionality that relates the cost in dollars,y, to the number of pounds of boudin,x?

Answers

Given :

The cost of 5 pounds of boudin is $26.

To Find :

The constant of proportionality that relates the cost in dollars y, to the number of pounds of boudin x .

Solution :

Price per boudin in terms of x and y is :

[tex]P=\dfrac{y}{x}[/tex]

Price per boudin by given data :

[tex]P=\dfrac{26}{5}[/tex]

So,

[tex]\dfrac{y}{x}=\dfrac{26}{5}\\\\y = \dfrac{26}{5}x[/tex]

Therefore, the constant of proportionality is [tex]\dfrac{26}{5}[/tex].

Hence, this is the required solution.

What is the vertical intercept (or y-intercept) of the line below?
(0,6)
(-3,0)

Answers

Answer:

-6 ?

Step-by-step explanation:

a. {[(-15+5)×2+8]-32÷8}-(-7)

b. (5-2)³×2+[-4+(-7)]÷(-2+4)²

c. {-10-[12+(-3)²] + 3³}+3³}÷(-3)

I'll mark you brainliest if you solve these with explanation​

Answers

Answer:

Step-by-step explanation:

The answers are:

A)  {[(-15+5)×2+8]-32÷8}-(-7)

Calculate within parentheses: -16 - (-7)

Add and subtract (left to right): -16 - (-7) = -9

B) (5-2)³×2+[-4+(-7)]÷(-2+4)²

To solve this problem, you need to follow the steps of order of operations.

Calculate within parentheses: [tex]3^{3}[/tex] * 2 + [-4 + (-7)] ÷ (-2+4)²

Calculate within parentheses: [tex]3^{3}[/tex] *2 -11 ÷ (-2+4)²

Calculate within parentheses: 27 × 2 - 11 ÷ (-2+4)²

Calculate exponents: 27 × 2 -11 ÷ 4

Multiply and divide: 54 - 11 ÷ 4

Multiply and divide: 54 - [tex]\frac{11}{4}[/tex]

Add and Subtract:  [tex]\frac{205}{4}[/tex]

Convert improper fractions to mixed numbers: [tex]\frac{205}{4}[/tex] = [tex]51\frac{1}{4}[/tex]

C) {-10-[12+(-3)²] + 3³}+3³}÷(-3)

Calculate within parentheses: -4 ÷ (-3)

Multiply and divide:  [tex]\frac{-4}{(-3)}[/tex]

Simplify and convert improper fraction to a mixed number:

[tex]\frac{-4}{(-3)}[/tex] = [tex]\frac{4}{3}[/tex] = [tex]1\frac{1}{3}[/tex]

Hope this helped! If so, please mark brainliest.

plz help what is 6 x 18 plz show work thank

Answers

Answer:

the answer is 108.

Step-by-step explanation:

6 x 18 = 108

6 x 18= 108

4
1 8
X 6
108

Hope it helps :)

The 7th-grade class took a survey on their favorite ice cream. 58 people said they prefer vanilla, while 28 people said they prefer chocolate. Write a ratio comparing chocolate to vanilla. (Write it in a :b form)

Answers

Since 58 for vanilla and 28 for chocolate
The ratio would be 28:58
You can simplify it even further
to 14:29

Swanton Savings Bank pays 5.5% interest compounded quarterly on a regular savings account. John deposited $4400 in regular savings accounts for 1.5 years. How much interest did he earn?

Answers

Answer:

$375.71

Step-by-step explanation:

Given that:

Rate of interest = 5.5% compounded quarterly

Time = 1.5 years

Principal amount deposited in savings account = 1.5 years

To find:

Interest earned = ?

Solution:

Formula for amount with interest compounding quarterly:

[tex]A = P(1+\dfrac{r}{4\times 100})^{4t}[/tex]

Putting the values:

[tex]A = 4400(1+\dfrac{5.5}{4\times 100})^{4\times 1.5}\\\Rightarrow 4400\times 1.01375^6\\\Rightarrow 4400\times 1.085\\\Rightarrow \$ \bold{4775.71}[/tex]

Interest earned = Amount - Principal

Interest earned = 4775.70 - 4400 = $375.71

Identify the angles that each have a measure of 88

Answers

Answer: 86 and 4

Step-by-step explanation:

Hopefully this is the right answer!

Answer:

the answer for this question above is 1

Step-by-step explanation:


If the perimeter of ABC = 9.7cm,
E is the midpoint of AC, and DE||BC, find the length of DE.

Answers

Answer:

3.2333 (repeated)

3.23 (rounded

Step-by-step explanation:

ADE is 1/3 of ABC so we just divide the perimeter by 3

9.7cm / 3 = 3.2333

This rounds to: 3.23

solve by factoring x^2 -9 =0​

Answers

x = 3 and -3

X^2 - 9 = 0
(X+3)(X-3) = 0

X + 3 = 0
X = -3

X - 3 = 0
X = 3

Find the area of the shape above. Will give brainliest answer.

Answers

Answer: 3/5

Step-by-step explanation:

A=a+b/2h

Since the bottom width = 4 and the top length =2 and the length = 5

2+4=6

2(5)=10

so 6/10 = 3/5

Can someone help me please

Answers

x=23 would be the answer

There are 40 DuBois students going on a field trip. Each one pays their teacher $8.75 to cover admission to the museum and lunch. Admission for the students is $120 and each one gets an equal amount to spend on lunch. How much will each DuBois student be able to spend on lunch? *

Answers

Given :

There are 40 DuBois students going on a field trip.

Each one pays their teacher $8.75 to cover admission to the museum and lunch.

Admission for the students is $120 and each one gets an equal amount to spend on lunch.

To Find :

How much will each DuBois student be able to spend on lunch.

Solution :

Amount they paid for admission :

[tex]P =\$ \dfrac{120}{40}\\\\P=\$3[/tex]

Now, amount of money to spend on lunch is $8.75 - $3 = $5.75 .

Hence, this is the required solution.

If I parallel m, find the value of x and y

Answers

Answer:

x=24 y=9.8

Step-by-step explanation:

5y-23=3x

47+3x+2x+13=180

please help
asap
someone please help

the measure of angle of a quadrilateral is 50 120 and 100 degree. find the measure of fourth angle​

Answers

Answer:

The measure of the fourth angle is 90°.

Step-by-step explanation:

Let x be the measure of the fourth angle.

Given measures of the other angles : 50°,120° & 100°.

Knowledge Required :

The sum of all the interior angles of a quadrilateral is 360°.

Then,

50°+120°+100°+x=360°

⇒x = 360°-270°

⇒x = 90°

.°. x = 90°

Hence, the measure of the fourth angle is 90°.

Which is the "The sum of 4 times a number and 7" as an algebraic expression?
A)4(x+7)
B)(4.7)+x
C)4-7+x
D)4x+7

Answers

The Answer is A because you are multiplying 4 into the a number (x) and 7 in the parentheses
ITs a because u is timing the number that is reduce

I NEED THIS ANSWER I DONT UNDERSTAND PLEASE HELP

Answers

Answer:

A. 904.3 cm³

Step-by-step explanation:

Radius of the ice cream cone (r) = 6 cm

Height of the ice cream cone (h) = 12 cm

Radius of the shape of hemisphere formed by the half scope of ice cream = 6 cm

Total volume of ice cream = volume of cone + volume of hemisphere

= [tex] (\frac{1}{3} \pi r^2 h) + (\frac{2}{3} \pi r^3) [/tex]

Plug in the values

[tex] (\frac{1}{3} \times 3.14 \times 6^2 \times 12) + (\frac{2}{3} \times 3.14 \times 6^3) [/tex]

[tex] (\frac{1}{3} \times 3.14 \times 36 \times 12) + (\frac{2}{3} \times 3.14 \times 216) [/tex]

[tex] = (1 \times 3.14 \times 12 \times 12) + (2 \times 3.14 \times 72) [/tex]

[tex] = (452.16) + (452.16) [/tex]

Total volume of ice cream = 904.3 cm³ (nearest tenth)

What are the ratios of sine, cosine, and tangent for
angle Y?
sin(Y) = XZ; cos(Y) = Y? ; tan(Y) = YZ
XY
XZ
XY
sin(Y) = XY; cos(Y) = XZ; tan(Y) = YZ
XZ
XY
XZ
O sin(Y) = x7; COS(Y) = x7; tan(Y) = {Z
sin(Y) = YZ ; cos(Y) = XZ; tan(Y) = XZ
XY
XY
YZ

Answers

Answer:

C!!!!

Step-by-step explanation:

Answer: The answer is C

Step-by-step explanation:

Got it write on edge

PLEASE HELP ME I HAVE A SICKNESS CALLED IM DUMB AND I REALLY NEED HELP WITH THESE QUETIONS. IF YOU CAN ANSWER THEM ALL ILL GIVE BRAINLIEST AND 50 POINTS AND PLSS DONT DO IT FOR THE POINTS.PLSSS

Answers

Step-by-step explanation:

the answer in the image

The supply curve for steel is 90x−y=−50. The demand curve is 10x+y=200. x is quantity in thousands of metric tons y is price in dollars per metric ton What are the values for quantity and price at the intersection point of the two curves? To find the values, first use addition to combine the left sides and to combine the right sides of the two equations.

Answers

Answer:

The intersection point of the two curves is [tex](x,y) = \left(\frac{3}{2}\,kton.m, 185\,USD \right)[/tex].

Step-by-step explanation:

From statement we get the following equations:

Supply curve

[tex]90\cdot x - y = -50[/tex] (1)

Demand curve

[tex]10\cdot x + y = 200[/tex] (2)

Where:

[tex]x[/tex] - Quantity, measured in thousands of metric tons.

[tex]y[/tex] - Price, measured in US dollars per metric tons.

If we add both equations, then we find that quantity is:

[tex](90\cdot x -y )+(10\cdot x +y) = -50+200[/tex]

[tex]100\cdot x = 150[/tex]

[tex]x = \frac{3}{2}\,kton.m[/tex]

Then, we finally find the price by substituting on (2):

[tex]y = 200-10\cdot \left(\frac{3}{2} \right)[/tex]

[tex]y = 200-15[/tex]

[tex]y = 185\,USD[/tex]

The intersection point of the two curves is [tex](x,y) = \left(\frac{3}{2}\,kton.m, 185\,USD \right)[/tex].

13. Jennifer has $900 in a savings account at the beginning of the summer. She wants to have at least $250 in the account by the end of November. She withdraws $35 each week for food and shopping. If w represents the number of weeks, which of the following inequalities could be used to solve for the amount of money she may have by the end of summer?

Answers

Answer:

455

Step-by-step explanation:

900-(35×13)=455

Lines j and k are perpendicular. The slope of line j is given. Find the slope of line k. m = 4/5

Answers

Answer:

A

Step-by-step explanation:

Given a line with slope m then the slope of a line perpendicular to it is

[tex]m_{perpendicular}[/tex] = - [tex]\frac{1}{m}[/tex] = - [tex]\frac{1}{\frac{4}{5} }[/tex] = - [tex]\frac{5}{4}[/tex] → A

One lap around a dirt track i 1/3 mile. It takes bryce 1/9 hour to ride one lap. what is bryce's unit rate, in miles, around the track?

Answers

Answer:

3 mph

Step-by-step explanation:

To make 1/9 hour to 1 hour, you need to x9.

You can use this way:

Let the speed be x

(1/3)/(1/9)=x/1

1/9x=1/3

x=3

3mph

Hope is this useful :D

Evaluate x/4 for x=48

Answers

Hey there!

If x = 48

48/4 = 12 because 12 • 4 equals 48

Answer: 12

Good luck on your assignment and enjoy your day!

~LoveYourselfFirst:)

x=8

Step-by-step explanation:

{x}{4}x=48

{x^2}{4}=48

{4x^2}{4}=48\

The $414 price charges a store for a small refrigerator consists of the refrigerator's original cost to the store plus a profit 15% what was the refrigerator's original cost to the store?

Answers

Answer:

$351.9

Step-by-step explanation:

the question is not formatted well

A store charges $414 for a small refrigerator, the price consists of the refrigerator's original cost to the store plus a profit 15% what was the refrigerator's original cost to the store?

Step one:

given data

we are told that the price of the small refrigerator contains both the original price and 15% profit

mathematically

$414= original price + 15% profit

Step two:

let us solve for 15% of 414

=(15/100)*414

=0.15*414

=$62.1

This shows that the profit is $62.1

let the original price be x

$414= x + 62.1

solve for x

x=414-62.1

x=351.9

The refrigerator's original cost to the store $351.9

What does W equal? I’m lost

Answers

Answer:

it looks like it would be 69% like the one below it

Step-by-step explanation:

What is the factor to this as a expression?

Answers

Answer:

The expression is not factorable with rational numbers

Step-by-step explanation:

PLEASE HELP ME
Write the equation from each line

Answers

Answer:

1) y = 2x - 2

2) y = - x - 1

3) y = 2/3 x + 1

4) y = 4x - 4

5) y = 3x + 6

6) y = -3x -3

7) y = 3 - x

8) y = 4 - 4x

Use the chart to help u ;-;"

Answers

It would be about 32,120 All you would do is take how much they eat a day then multiply by 365
The answer is 32120 pounds

Choose Yes or No to tell if the number 103 will make each equation true. 6.01 × □ = 601 No 0.305 × □ = 305 No 0.54 × □ = 540 No 0.097 × □ = 970 Yes

Answers

Answer:

Kindly check explanation

Step-by-step explanation:

Given the equations :

6.01 × □ = 601 No 0.305 × □ = 305 No 0.54 × □ = 540 No 0.097 × □ = 970 Yes

Will the number 103 will make each true?

601 / 6.01= 100 (No, not true)

305 / 0.305 = 1000 (No, not true)

540 / 0.54 = 100 (No, not true)

970 / 0.097 = 10000 (No, not true)

Hence, multiplying each of the equation above by 103 will not make any of the equations true.

Other Questions
A company uses the weighted average method for inventory costing. During a period, Department B finished and transferred 67,000 units to Department C. Also in Department B during the period, 18,500 units were started but brought only to a stage of being 60% completed. The number of equivalent units produced by Department B during the period was: You take the vital signs for a patient who does not appear to be in any distress and denies complaints of pain or shortness of breath. His oxygen saturation is 87% on room air. You see in his previously charted vitals the saturation was 86%. You have two other patients that have requested your attention including one who has been waiting for a shower. What will you do next? In the equation 3x+7=15, the number 7 is a. 13. Are ALL genes expressed in ALL cell types? Why does sin540.809cos36 in a right triangle?A 54 and 36 are complementary angles in the same right triangle. sin54 is the ratio of the opposite leg and the hypotenuse and cos36 is the ratio of the adjacent leg and the hypotenuse. In this triangle, the opposite leg of 54 and the adjacent leg of 36 are the same leg in the triangle.B 54 and 36 are complementary angles in the same right triangle. sin54 is the ratio of the adjacent leg and the hypotenuse and cos36 is the ratio of the opposite leg and the hypotenuse. In this triangle, the adjacent leg of 54 and the opposite leg of 36 are the same leg in the triangle.C 54 and 36 are supplementary in the same right triangle. sin54 is the ratio of the opposite leg and the hypotenuse and cos36 is the ratio of the adjacent leg and the hypotenuse. In this triangle, the opposite leg of 54 and adjacent leg of 36 are the same leg in the triangle.D 54 and 36 are supplementary in the same right triangle. sin54 is the ratio of the adjacent leg and the hypotenuse and cos36 is the ratio of the opposite leg and the hypotenuse. In this triangle, the adjacent leg of 54 and opposite leg of 36 are the same leg in the triangle. Compare the mass of samples A and B to determing if acidity affects the rate of weathering. Was there a measureable difference between the two treatments. yes, no, unable to determine. The limestone in the vinegar demonstrated a ___________ rate of weathering than the limestone in the water. faster or slower. Based on the data, it can be said that, acicidity has no effect on weathering, acidity affects the rate of weathering, the affect of acidity cannot be determined. PLEASE HELP Rpondez avec une expression ngative.1. Tu attends quelqu'un?2. Il mange encore?3. Vous prenez quelque chose?4. Tu regardes toujours la tl?5. Les enfants font la lessive ou la vaisselle? (Laundry /dishes)6. Nous prparons encore le dner?7. Tu attends quelque fois tes copains aprs les cours?8. Simone achte une jupe et un chemisier?9. Elles vont mercredi aprs-midi?10. Nous allons faire tout?11. Tu vas aller la piscine ou la plage?12. Grard invite tout le monde?13. lise coute la prof?14. Tout va bien? what is gastrointestinal tract? choose twocycle *A) repeatsB)uncertainC)predictableD)unknown What is the speed in kilometers per hour of a 27.78m/s baseball pitch? What is the answer to this question? will get awarded with brainliest (4a2+2)(6a2-a+2) hi counting down the days of thanksgiving, so here's a little treat. DAY 15! Read the excerpt from "How I Learned English." Someone said "shin" again, There was a wild stamping of hands on the ground, A kicking of feet, and the fit Of laughter overtook me too, And that was important, as important As Joe Barone asking me how I was The descriptions in this excerpt help readers imagine what is lady gaga`s 5th studio album and all the songs on itBRAINLIEST ANSWER!!! SOMEONE PLEASE HELP MEHow does Dickens present the Ghost of Christmas Past?(analyse and include quotations) Choose the correct answer to 8,817/43.25 R2205205 R2204 R45 A dog jumps 0.80 m to catch a treat. The dog's displacement vector is shown below.Which diagram shows the vertical and horizontal components of the dog's displacement?Choose 1 answer: An atom of an element has 2 electrons in the 1st energy level and 5 on the 2nd energy level what element is it ? What mass of NO2 can be produced when 678 mL of oxygen at STP and 256 grams of NH3 are mixed together? (first to answer gets brainiest)The artifacts found in Catal Huyuk's temples included___.A. potteryB. animal headsC. artworkD. all of the above